Prove or disaprove the statements

  • MHB
  • Thread starter mathmari
  • Start date
In summary, we have proven that for a continuous function $f:\mathbf R\to \mathbf R$, the image of a closed interval $[a,b]$ is again a closed interval $[x,y]$. This means that if $A$ is bounded, then $f(A)$ is also bounded.
  • #1
mathmari
Gold Member
MHB
5,049
7
Hey! :eek:

Let $f : \mathbb{R} \rightarrow \mathbb{R}$ be continuous and $A,B\subseteq \mathbb{R}$. Prove or disprove the following:
  1. $A$ closed $\Rightarrow$ $f(A)$ closed
  2. $B$ closed $\Rightarrow$ $f^{-1}(B)$ closed
  3. $A$ bounded $\Rightarrow$ $f(A)$ bounded
  4. $B$ bounded $\Rightarrow$ $f^{-1}(B)$ bounded
I have done the following:

We have that $D$ is closed $\iff$ for each convergent sequence $(x_n)$ in $D$ it holds that $\displaystyle{\lim_{n\rightarrow \infty} x_n\in D}$.

We also have that $f: D\rightarrow \mathbb{R}$ is bounded $\iff$ $f(D)$ is bounded. This is equivalent to $\exists c\geq 0 \ \ \forall x\in D : |f(x)|\leq c$.

  1. What can we do in this case? (Wondering)
    $$$$
  2. Let $(x_n)$ be a convergent sequence in $f^{-1}(B)$. We want to show that $\displaystyle{\lim_{n\rightarrow \infty}x_n\in f^{-1}(B)}$.
    Since $f$ is continuous, we have that $\displaystyle{f\left (\lim_{n\rightarrow \infty} x_n\right )=\lim_{n\rightarrow \infty} f(x_n)}$.
    Since $(x_n)\in f^{-1}(B)$ and $f$ is continuous we have that $f(x_n)\in B$. Is this correct? (Wondering)
    Since $B$ is closed we have that $\displaystyle{\lim_{n\rightarrow \infty}f(x_n)\in B}$.
    Therefore, we have that $\displaystyle{f\left (\lim_{n\rightarrow \infty}x_n\right )\in B}$. From that it implies that $\displaystyle{\lim_{n\rightarrow \infty}x_n\in f^{-1}(B)}$. This means that $f^{-1}(B)$ is closed.
    Is this correct? (Wondering)
    $$$$
  3. Since $A$ is bounded, we have that it is upper and under bounded. There are $x,y$ such that $$x\leq a \leq y , \ \ \text{ for all } a\in A$$ Since $f$ is continuous we have that $$f(x)\leq f(a) \leq f(y) , \ \ \forall a\in A$$
    This means that in $A$ the function $f$ is bounded.
    Therefore, $f(A)$ is bounded.
    Is this correct? (Wondering)
    $$$$
  4. What can we do in this case? (Wondering)
 
Last edited by a moderator:
Physics news on Phys.org
  • #2
mathmari said:
Hey! :eek:

Let $f : \mathbb{R} \rightarrow \mathbb{R}$ be continuous and $A,B\subseteq \mathbb{R}$. Prove or disprove the following:
  1. $A$ closed $\Rightarrow$ $f(A)$ closed
  2. $B$ closed $\Rightarrow$ $f^{-1}(B)$ closed
  3. $A$ bounded $\Rightarrow$ $f(A)$ bounded
  4. $B$ bounded $\Rightarrow$ $f^{-1}(B)$ bounded
I have done the following:

We have that $D$ is closed $\iff$ for each convergent sequence $(x_n)$ in $D$ it holds that $\displaystyle{\lim_{n\rightarrow \infty} x_n\in D}$.

We also have that $f: D\rightarrow \mathbb{R}$ is bounded $\iff$ $f(D)$ is bounded. This is equivalent to $\exists c\geq 0 \ \ \forall x\in D : |f(x)|\leq c$.

  1. What can we do in this case? (Wondering)
    $$$$

Consider $f(x)=\tan^{-1}(x)$. Then $f(\mathbf R)=(-\pi/2, \pi/2)$. Thus $(1)$ is not true.

mathmari said:
  1. Let $(x_n)$ be a convergent sequence in $f^{-1}(B)$. We want to show that $\displaystyle{\lim_{n\rightarrow \infty}x_n\in f^{-1}(B)}$.
    Since $f$ is continuous, we have that $\displaystyle{f\left (\lim_{n\rightarrow \infty} x_n\right )=\lim_{n\rightarrow \infty} f(x_n)}$.
    Since $(x_n)\in f^{-1}(B)$ and $f$ is continuous we have that $f(x_n)\in B$. Is this correct? (Wondering)
    Since $B$ is closed we have that $\displaystyle{\lim_{n\rightarrow \infty}f(x_n)\in B}$.
    Therefore, we have that $\displaystyle{f\left (\lim_{n\rightarrow \infty}x_n\right )\in B}$. From that it implies that $\displaystyle{\lim_{n\rightarrow \infty}x_n\in f^{-1}(B)}$. This means that $f^{-1}(B)$ is closed.
    Is this correct? (Wondering)
    $$$$
This is ok. Thus (2) is true.

mathmari said:
  1. Since $A$ is bounded, we have that it is upper and under bounded. There are $x,y$ such that $$x\leq a \leq y , \ \ \text{ for all } a\in A$$ Since $f$ is continuous we have that $$f(x)\leq f(a) \leq f(y) , \ \ \forall a\in A$$ (no, this may not be true).
  2. This means that in $A$ the function $f$ is bounded.
    Therefore, $f(A)$ is bounded.
    Is this correct? (Wondering)
    $$$$
One can show that the image of a closed interval under a continuous function $f:\mathbf R\to \mathbf R$ is again a alosed interval.
From this it will follow that (3) is true.

mathmari said:
  1. What can we do in this case? (Wondering)

As for (4), again take $f=\tan^{-1}$ and see that $f^{-1}(-\pi/2, \pi/2)=\mathbf R$. Thus (4) is false.
 
  • #3
caffeinemachine said:
One can show that the image of a closed interval under a continuous function $f:\mathbf R\to \mathbf R$ is again a alosed interval.

Isn't this the case 1., that we disaproved? (Wondering)
 
  • #4
Do you mean the following?

Since $A$ is bounded, there exists $b=\sup A$ and $a=\inf A$. So, $A\subseteq [a,b]$.
Since $f$ is continuous (or do we not need the continuity here ? ) we get that $f(A)\subseteq f([a,b])$.
$[a,b]$ is closed and $f$ is continuous, therefore $f([a,b])$ is closed. How could we prove that? (Wondering)
Is $f(A)$ then bounded because it is a subset of a closed interval? (Wondering)
 
  • #5
mathmari said:
Do you mean the following?

Since $A$ is bounded, there exists $b=\sup A$ and $a=\inf A$. So, $A\subseteq [a,b]$.
Since $f$ is continuous (or do we not need the continuity here ? ) we get that $f(A)\subseteq f([a,b])$.
$[a,b]$ is closed and $f$ is continuous, therefore $f([a,b])$ is closed. How could we prove that? (Wondering)
Is $f(A)$ then bounded because it is a subset of a closed interval? (Wondering)

In case I caused any confusion, by a closed interval I mean a subset of $\mathbf R$ of the form $[a, b]$, where $a<b$ are real numbers.

The statement I made was that if $f:\mathbf R\to \mathbf R$ is a continuous function, then $f$ maps a closed interval to a closed interval.

A one line proof of this would be to invoke the fact that any continuous image of a compact-connected topological space is again compact-connected.

Here is a more basic proof using sequences.

We want to show that $B:=f([a, b])$ is a closed interval. First we show that $B$ is bounded above. Suppose not. Then for each natural number $N$, there is $x_n\in [a, b]$ such that $f(x_n)>N$. Now by Bolzano-Weierstrass, the sequence $(x_n)$ has a convergent subsequence, say $(x_{n_k})$, which converges to say $x$. Thus $f(x_{n_k})\to f(x)$. But this means $f(x_{n_k})$ is bounded which is a contradiction. Similarly one can prove that $B$ is bounded below.

Let $x=\inf(B)$ and $y=\sup(B)$. Again, an application of Bolzano-Weierstrass shows that both $x$ and $y$ are in $B$. Now by intermediate value theorem it is clear that $B=[x, y]$.
 
Last edited:
  • #6
caffeinemachine said:
We want to show that $B:=f([a, b])$ is a closed interval. First we show that $B$ is bounded above. Suppose not. Then for each natural number $N$, there is $x_n\in [a, b]$ such that $f(x_n)>N$. Now by Bolzano-Weierstrass, the sequence $(x_n)$ has a convergent subsequence, say $(x_{n_k})$, which converges to say $x$. Thus $f(x_{n_k})\to f(x)$, giving $N_k\to f(x)$. But this means $N_k$ is bounded which is a contradiction. Similarly one can prove that $B$ is bounded below.

I haven't really understood the part with $N_k$. What exactly is $N_k$ ? (Wondering)
 
  • #7
mathmari said:
I haven't really understood the part with $N_k$. What exactly is $N_k$ ? (Wondering)
I have edited my post. (Actually, writing $N_k\to f(x)$ was not correct.)

To elaborate a bit, a subsequence of $(x_n)_{n\geq 1}$ is a sequence $(x_{n_k})_{k\geq 1}$, where $(n_k)_{k\geq 1}$ is a strictly increasing sequence of naturals.

Does that clear the confusion?
 
  • #8
So, we use the fact that a convergent sequence is bounded, right? (Wondering)
Having that $f(x_{n_k})$ is bounded, means that $f(x_n)$ must also be bounded? (Wondering)
caffeinemachine said:
Let $x=\inf(B)$ and $y=\sup(B)$. Again, an application of Bolzano-Weierstrass shows that both $x$ and $y$ are in $B$. Now by intermediate value theorem it is clear that $B=[x, y]$.

Let w be a number between x and y, then there must be at least one value c within [a, b] such that f(c) = w, right? How does it follow from that that $B=[x,y]$ ? I got stuck right now... (Wondering) In general, it holds that $B\subseteq [x,y]$, or not? Since $x,y\in B$, it follows that $B=[x,y]$, right? (Wondering)
 
Last edited by a moderator:
  • #9
mathmari said:
So, we use the fact that a convergent sequence is bounded, right? (Wondering)
Yes.

mathmari said:
Having that $f(x_{n_k})$ is bounded, means that $f(x_n)$ must also be bounded? (Wondering)
No. You can come up with easy counterexamples to this.

mathmari said:
Let w be a number between x and y, then there must be at least one value c within [a, b] such that f(c) = w, right?
Yes.

mathmari said:
How does it follow from that that $B=[x,y]$ ? I got stuck right now... (Wondering)

In general, it holds that $B\subseteq [x,y]$, or not? Since $x,y\in B$, it follows that $B=[x,y]$, right? (Wondering)

We defined $x=\inf(B)$ and $y=\sup(B)$. Thus we have $B\subseteq [x, y]$. The reverse inequality is by the intermediate value theorem.
 
  • #10
caffeinemachine said:
No. You can come up with easy counterexamples to this.

caffeinemachine said:
We want to show that $B:=f([a, b])$ is a closed interval. First we show that $B$ is bounded above. Suppose not. Then for each natural number $N$, there is $x_n\in [a, b]$ such that $f(x_n)>N$. Now by Bolzano-Weierstrass, the sequence $(x_n)$ has a convergent subsequence, say $(x_{n_k})$, which converges to say $x$. Thus $f(x_{n_k})\to f(x)$. But this means $f(x_{n_k})$ is bounded which is a contradiction.

But how do we get the contradiction?

Do we maybe have to suppose that for each sequence $x_n\in [a,b]$ there is a natural number $N$ such that $f(x_n)>N$ and not that for each natural number $N$, there is $x_n\in [a, b]$ such that $f(x_n)>N$? (Wondering)
 
Last edited by a moderator:
  • #11
caffeinemachine said:
Consider $f(x)=\tan^{-1}(x)$. Then $f(\mathbf R)=(-\pi/2, \pi/2)$. Thus $(1)$ is not true.

How can we show that $f(\mathbf R)=(-\pi/2, \pi/2)$ ? (Wondering)
 
  • #12
mathmari said:
3. $A$ bounded $\Rightarrow$ $f(A)$ bounded

Could we maybe show it also as follows?

We suppose that $f(A)$ is not bounded.

There is also a sequence $\{x_n\}_{n=1}^{\infty}$ in $f(A)$ with $\lim x_n=\infty$. Or do we not get that when $f(A)$ is not bounded? (Wondering)

Since $\{x_n\}_{n=1}^{\infty} \in f(A)$ it follows that $\{f^{-1}(x_n)\}_{n=1}^{\infty} \in A$, or not? (Wondering)

$\{f^{-1}(x_n)\}_{n=1}^{\infty}$ is therefore, bounded, right? (Wondering)

From Bolzano-Weierstrass there is a convergent subsequence, let $\{f^{-1}(x_{n_k})\}_{k=1}^{\infty}$ with $f^{-1}(x_{n_k})\rightarrow y\in \mathbb{R}$.

Since $f$ is continuous, we have that $f\left (f^{-1}(x_{n_k})\right )\rightarrow f(y)\in \mathbb{R}$.

That means that $x_{n_k}\rightarrow f(y)$.

But how could we get a contradiction? (Wondering)
 
Last edited by a moderator:
  • #13
mathmari said:
Could we maybe show it also as follows?

We suppose that $f(A)$ is not bounded.

There is also a sequence $\{x_n\}_{n=1}^{\infty}$ in $f(A)$ with $\lim x_n=\infty$. Or do we not get that when $f(A)$ is not bounded? (Wondering)
It could be that there is no sequence in $f(A)$ whose limit is infinity, given that $f(A)$ is unbounded. But what is true for sure is that that is a sequence whose limit is either $+\infty$ or $-\infty$. We may WLOG assume that the limit is $+\infty$.

mathmari said:
Since $\{x_n\}_{n=1}^{\infty} \in f(A)$ it follows that $\{f^{-1}(x_n)\}_{n=1}^{\infty} \in A$, or not? (Wondering)

Note that $f^{-1}(x_n)$ is a subset of of $\mathbf R$. So $\{f^{-1}(x_n)\}$ is not a sequence in $\mathbf R$.
So the way to go is by choose $y_n\in f^{-1}(x_n)$ for each $n$. Then you get a sequence $\{y_n\}$ in $\mathbf R$.

Now from Bolzano-Weierstrass there is a convergent subsequence $\{y_{n_k}\}_{k=1}^{\infty}$ with $y_{n_k}\rightarrow y\in \mathbb{R}$.

Since $f$ is continuous, we have that $x_n=f(y_n)\rightarrow f(y)\in \mathbb{R}$.

That means that $x_{n_k}\rightarrow f(y)$. The contradiction is that since $x_{n_k}$ is a subsequence of $\{x_n\}$, we also have $x_{n_k}\to \infty$.
 
  • #14
I see! Thank you very much! (Smile)
 

Related to Prove or disaprove the statements

1. Can you provide evidence to support or refute the statement?

Yes, as a scientist, it is important to gather and analyze data in order to support or disprove a statement. This can include conducting experiments, collecting observations, or reviewing existing research.

2. How do you determine the validity of a statement?

The validity of a statement can be determined by examining the evidence and evaluating its reliability and relevance. Additionally, the statement should be consistent with established scientific principles and theories.

3. What role does statistics play in proving or disproving a statement?

Statistics are crucial in analyzing data and drawing conclusions. By using statistical tests, scientists can determine the likelihood of their findings being due to chance, and therefore support or refute a statement with more confidence.

4. Can one study be enough to prove or disprove a statement?

It is generally not enough to rely on a single study to prove or disprove a statement. In order to increase confidence in the results, it is important to replicate the study and gather data from multiple sources.

5. What is the difference between proving and disproving a statement?

Proving a statement means providing evidence that supports its truth, while disproving a statement means providing evidence that contradicts its truth. Both processes involve gathering and analyzing data, but have different outcomes.

Similar threads

Replies
7
Views
2K
Replies
23
Views
2K
  • Topology and Analysis
Replies
11
Views
957
  • Topology and Analysis
Replies
4
Views
2K
Replies
2
Views
401
Replies
40
Views
5K
  • Topology and Analysis
2
Replies
38
Views
3K
Replies
3
Views
233
  • Topology and Analysis
Replies
11
Views
1K
  • Topology and Analysis
2
Replies
44
Views
5K
Back
Top